Vous êtes sur la page 1sur 26

Unit I Laplace Transform

Laplace Transform
The Laplace transform is an integral transform that transforms a real valued
function f of some non-negative variable, say t into a function F(s) for all s for
which the improper integral

0
st
dt ) t ( f e
converges.
Definition 1.1 Let f (t) be a given real valued function that is
defined for
all t 0. The function defined by

0
) ( dt t f e
st
for all s for which this improper integral converges is
called the
Laplace transform of f (t) and will be denoted by
F (s) = L (f) =

0
) ( dt t f e
st
The operation which yields F (s) from a given real valued f (t) is also called the
Laplace transform of f (t) while f (t) is called the inverse transform of F (s)
and will be denoted by
f (t) = ) F ( L
1
Example 1 Let f (t) = 1 for t 0. Then find F (s).
Solution Applying definition 1.1 we get:
L (f) = L (1) =



b
0
st
dt e
b
im
=
0
b
st
s
e
b
im

'

'

s
e
b
im
s
1
t b

=
s
1
for s
> 0.
Therefore, L (1) =
s
1
for s > 0.
Remark: Let f (t) = k for any scalar k. Then L (f) =
s
k
for s > 0.
Example 2 Let

'

>

1 , 0
1 0 , 4
) (
t for
t for
t f . Then find F (s).
Prepared by Tekleyohannes Negussie
83
Unit I Laplace Transform
Solution L (f) = F (s) = dt e
b
im
b
t f
st



0
) (

= dt e
st


1
0
4 = ( )
s
e
s

1
4
.
Therefore, L (f) = ( )
s
e
s

1
4
for s > 0.
Example 3 Let f (t) = t for t 0. Then find F (s).
Solution Applying definition 1.1 we get:
L (f) = F (s) =



b
0
st
dt e t
b
im

Now applying integration by parts we get:



b
0
st
dt e t
b
im
=
0
2
b
st
st
s
e
e
s
t
b
im

'

=
2
s
1
for s > 0.
Therefore, L (f) =
2
1
s
for s > 0.
Example 4 Prove that for any natural number n, L (
n
t ) =
1
!
+ n
s
n
for t 0.
Solution We need to proceed by applying the principle of mathematical
induction on n
i) For n = 1 it follows from example 3.
ii) Assume that it holds true for n = k, i.e.
L (
k
t ) =
1
!
+ k
s
k
where t 0
iii) We need to show that it holds true for n = k + 1. i.e. L (
1 + k
t ) =
2
! ) 1 (
+
+
k
s
k

for t 0.
Now by the definition of the Laplace transform
L (
1 + k
t ) =

+

b
t s k
dt e t
b
im
0
1

Applying integration by parts we get:


u =
1 + k
t
and
' v
=
t s
e

dt
then ' u =
k
t k ) 1 ( + dt and v =
t s
e
s

1
.
Hence,

+

b
t s k
dt e t
b
im
0
1

=
1
1
]
1

,
_

+
+

'




+ b
t s
k
t s
k
dt e t
s
k
b
e
s
t
b
im
0
1
1
0

Prepared by Tekleyohannes Negussie


84
Unit I Laplace Transform
=

,
_

+
b
t s
k
dt e t
b
im
s
k
0
1

=

,
_

,
_

+
+1
! 1
k
s
k
s
k
, by our
assumption
=
2
! ) 1 (
+
+
k
s
k
for s > 0.
Thus, by the principle of mathematical induction it holds true for any natural
number n.
Therefore, L (
n
t ) =
1
!
+ n
s
n
for t 0.
Example 5 Let f (t) =
t
e

for t 0. Then find F (s).
Solution Applying definition 1.1 we get:
L (f) = L (
t
e

) =



b
0
t ) s (
dt e
b
im
=
0
b
t ) s (
s
e
b
im

'

'


+


s
1
s
e
b
im
b ) s (

=
s
1
for s > .
Therefore, L (
t
e

) =
s
1
for s > .
Theorem 1.1 (linearity of the Laplace
transform)
The Laplace transform is a linear operation, that
is for
any real valued functions f (t) and g (t) whose
Laplace
transform exists and any scalars and
L ( f + g) = L (f ) + L ( g)
Proof By definition 1.1
L ( f + g) = { }


b
t s
dt e
b
im t g t f
0
) ( ) (
=



b
t s
dt t f e
b
im
0
) ( +



b
t s
dt t g e
b
im
0
) (
= h L (f) + k L ( g).
Therefore, Laplace transform is a linear operation.
Example 6 Find the Laplace transform of the hyperbolic functions f (t) = cosh
t and
Prepared by Tekleyohannes Negussie
85
Unit I Laplace Transform
g (t) = sinh t for t 0.
Solution From theorem 1.1 and the result of example 2 we get:
L (f ) = L (cosh t) =



b
t s
dt e
b
im
0
) (
2
1

+

b
t s
dt e
b
im
0
) (
2
1


=
0
2
1
) ( ) ( b
s
e
s
e
b
im
t s t s

'

'


+
s
e
s
e
b
im
b s b s

) ( ) (
2
1

'

s s
1 1
2
1
=
2 2
s
s
for s
> .
Similarly, L (g) = L (sinh t) =



b
t s
dt e
b
im
0
) (
2
1

+

b
t s
dt e
b
im
0
) (
2
1


=
0
2
1
) ( ) ( b
s
e
s
e
b
im
t s t s

'

+
+

+

'

+
+

+
s
e
s
e
b
im
b s b s

) ( ) (
2
1

'

+
+
s s
1 1
2
1
=
2 2

s
for s
> .
Therefore, L (cosh t) =
2 2
s
s
and L (sinh t) =
2 2

s
for s > .
Example 7 Find the Laplace transform of functions f (t) = cos t and g (t) = sin
t for t 0.
Solution From theorem 1.1 and Eulers formula, where i = 1 we get:

t i
e

= cos t + i sin t
L (f ) = L (
t i
e

) =
i s
1
=
) ( ) (

i s i s
i s
+
+
=
2 2 2 2

+
+
+ s
i
s
s
On the other hand, L (cos t + i sin t) = L (cos t) + i L (sin t).
Now equating the real and the imaginary parts we get:
L (cos t ) =
2 2
+ s
s
and L (sin t) =
2 2

+ s
.
Therefore, L (cos t ) =
2 2
+ s
s
and L (sin t) =
2 2

+ s
for s > .
We can also derive these by using definition 1. 1 and integration by parts with
out going into the operations in complex numbers.
Example 8 Find L (f), where
Prepared by Tekleyohannes Negussie
86
Unit I Laplace Transform
i) f (t) = 3 cos 2t + 2 sin t
3
4t ii) f (t) = 4 cosh 5t 3 sinh 2t +
t
e

5
iii) f (t) = sin 2t cos 4t
Solutions By the Linearity of the Laplace transform we have
i) L (f) = 3 L (cos 2t) + 2 L (sin t) ) ( 4
3
t L =
4
3
2
+ s
s
+
1
2
2
+ s

4
) ! 3 ( 4
s
.
Therefore, L [3 cos 2t + 2 sin t
3
4t ] =
4
3
2
+ s
s
+
1
2
2
+ s

4
24
s
for s > 0.
ii) L (f) = 4 L (cosh 5t) 3 L (sinh 2t) + ) ( 5
t
e L

=
25
4
2
s
s

4
6
2
s
+
s
5
.
Therefore, L [4 cosh 5t 3 sinh 2t +
t
e

5 ] =
25
4
2
s
s

4
6
2
s
+
s
5
for
s > 0.
iii) First observe that:
sin 2t cos 4t = t t ) 4 2 ( sin
2
1
) 4 2 ( sin
2
1
+ + = t t 6 sin
2
1
2 sin
2
1
+
Then by the Linearity of the Laplace transform we get:
L [sin 2t cos 4t] = ] 6 [sin
2
1
] 2 [sin
2
1
t L t L + =

,
_

+
+

,
_

36
6
2
1
4
2
2
1
2 2
s s
.
Therefore, L [sin 2t cos 4t] =

,
_

+
+

,
_

36
3
4
1
2 2
s s
for s > 0.
Example 9 For any real valued functions f (t) and g (t) whose Laplace transform
exists and
any real numbers and , show that the inverse Laplace transform
is linear.
Solution From the linearity of the Laplace transform we get:
L ( f (t) + g (t)) = L (f ) + L ( g ) = F (s ) + F ( s )
Hence,
)) ( ) ( (
1
s F s F L +

= f (t) + g (t)) =
) (
1
) (
1
s G s F L L

+


.
Therefore, the inverse Laplace transform is linear.
Example 10 Find the inverse Laplace transforms of
i)
s 2
3
ii)
3
40
s
iii)
1
2
2
+
+
s
s
iv)
9
4
2

+
s
s
Solutions i) From the linearity of the inverse Laplace transform we get:
Prepared by Tekleyohannes Negussie
87
Unit I Laplace Transform

1
]
1

s
L
2
3 1
=
1
]
1

s
L
1 1
2
3
=
2
3
.
Therefore,
1
]
1

s
L
2
3 1
=
2
3
.
ii) Now
3
40
s
=
1
]
1

3
! 2
20
s
and
2
3
! 2 1
t
s
L

1
]
1

.
Hence,
1
]
1


3
40 1
s
L =
1
]
1


3
! 2 1
20
s
L =
2
20t .
Therefore,
1
]
1


3
40 1
s
L =
2
20t .
iii)
1
]
1

+
+
1
2
2
1
s
s
L =
1
]
1

1
2
1
s
s
L +
1
]
1

1
1
2
2
1
s
L = cos t + 2 sin t.
Therefore,
1
]
1

+
+
1
2
2
1
s
s
L = cos t + 2 sin t.
iv)
1
]
1

+
9
4
2
1
s
s
L =
1
]
1

9
2
1
s
s
L +
1
]
1

,
_


9
3
3
4
2
1
s
L = cosh 3t +
,
_

3
4
sinh
3t.
Therefore,
1
]
1

+
9
4
2
1
s
s
L = cosh 3t +
,
_

3
4
sinh 3t.
Example 11 Find f (t) if
i) F (s) =
s s 3
9
2
+
ii) F (s) =
9
) 1 ( 4
2

+
s
s

Solution By partial fraction reduction we get:
i)
s s 3
9
2
+
=
s
A
+
3 + s
B
=
s s
s B s A
3
) 3 (
2
+
+ +
A + B = 0 and 3 A = 9 A = 3
and B = 3
Thus, F (s) =
s
3

3
3
+ s
=

,
_

,
_

3
3
1 1 1 1
3
s s
L L = 3 3
t
e
3
.
Therefore, f (t) = 3 3
t
e
3
.
ii)
9
) 1 ( 4
2

+
s
s
=
3 s
A
+
3 + s
B
=
9
) 3 ( ) 3 (
2

+ +
s
s B s A

A + B = 4 and 3 (A B) = 4 A =
3
8
and B =
3
4
Thus, F (s) =
3
8

,
_

3
1
s
+
3
4

,
_

+ 3
1
s
=

,
_

,
_

3 3
4
3
1 1 1 1
3
8
s s
L L =
t
e
3
3
8
+
3
4
t
e
3
.
Prepared by Tekleyohannes Negussie
88
Unit I Laplace Transform
Therefore, f (t) =
t
e
3
3
8
+
3
4
t
e
3
.
Example 12 For any 0, the Gamma function () is defined by:
() =


0
1
dt t e
t
Show that ) (
a
t L =
1
) 1 (
+
+
a
s
a
for a > 0 and in particular (n + 1) = n!,
for any non-
negative integer n.
Solution Put x = st, so that dt =
s
dx
.
Thus, ) (
a
t L =
s
dx
s
x
e
b
im
b
a
x


,
_

= dx x e
b
im
s
b
a
x
a

+

0
1
1

=
1
) 1 (
+
+
a
s
a
.
Therefore, ) (
a
t L =
1
) 1 (
+
+
a
s
a
for a > 0.
Further more, for any natural number n
(n + 1) = dt e
b
im
b
n
t
t

=
0
b
n
t
t
e
b
im

,
_

+ n
dt e
b
im
b
n
t
t



0
1

= n dt e
b
im
b
n
t
t



0
1

= n (n)
Thus, (n + 1) = n (n) = n (n 1) (n 1) = n (n 1) (n 2) (n 2) = . . . =
n! (1) = n!
We summarize the Laplace Transforms of some functions for future reference in
the table below.
N
o
Function f
(t)
Laplace
transform
N
o
Function f
(t)
Laplace
transform
1 1
s
1
6
t
e

s
1
2
k for k
s
k
7
cos t
2 2
+ s
s
3 t
2
1
s
8
sin t
2 2

+ s
Prepared by Tekleyohannes Negussie
89
Unit I Laplace Transform
4
n
t
for
n = 0, 1,
2, . . .
1
!
+ n
s
n
9
cosh t
2 2
s
s
5
a
t
for a
0
1
) 1 (
+
+
a
s
a
10
sinh t
2 2

s
Existence of Laplace Transforms
Before we define the theorem that guarantees the existence of Laplace
transform, let us see the definition of piecewise continuity.
Definition 1.2 A function f (t) is piecewise continuous on a
finite
interval a t b if f (t) is defined on the interval and is
such that
the interval can be divided into finitely many subintervals ,
in
each of which f (t) is continuous and has finite limits as t
approaches either endpoint of the subintervals from the
interior.
By definition 1.2, it follows that finite jumps are the only discontinuities that a
piecewise
continuous function may have.
Theorem 1.2 (Existence of Laplace Transforms)
Let f (t) be a real valued function that is piecewise
continuous
on every finite interval in [0, ) and satisfies the
Prepared by Tekleyohannes Negussie
x
y
Figure 1.1 a piecewise continuous function
90
Unit I Laplace Transform
inequality
) t ( f
t
e M

for all t 0 and for some constants

and M.
Then there
is a unique Laplace transform of f (t) for all s >

.
Proof Since f (t) is piecewise continuous, ) t ( f e
st
is integrable over any finite
interval on
the t-axis. Thus, for s >

we have
) f ( L =



b
0
) t ( f
st
dt e
b
im



b
0
) t ( f
st
dt e
b
im



b
st t
dt e e M
b
im
0

=
s
M
for s >

.
Therefore, ) ( f L is finite, and hence it exists for s >

.
Example 13 Let f (t) =
t
1
for t > 0. Then show that L [f] doesnt exist.
Solution We prove this by showing that dt e
t
t s

0
1
diverges. For some c > 0,
dt e
t
t s

0
1
= dt e
t b
im
c
b
t s

1
0

+ dt e
t d
im
d
c
t s

To this end, let


u =
t s
e

and
' v
= dt
t
1
. Then du = dt
t s
e s

and v = t n .
Hence,
dt e
t b
im
c
b
t s

1
0

=
{ }
b
c
t s
t n e
b
im


+
0
+
dt t n e s
b
im
c
b
t s


+
0
.
But, { }
b
c
t s
t n e
b
im


+
0
= { } b n e c n e
b
im b s c s

+
0

=
c s
e
c n
{ } b n
b s
e
b
im


+
0
= for any c > 0.
Consequently, dt e
t
t s

0
1
diverges.
Therefore, the Laplace transform of f (t) =
t
1
for t > 0 doesnt exist.
Remark: If the Laplace transform of a given real valued function exists, then it
is unique.
Prepared by Tekleyohannes Negussie
91
Unit I Laplace Transform
Conversely, if two functions have the same Laplace transform, then these
functions
are equal over any interval of positive length (i.e. they differ at various
isolated points).
Exercises 1.1
In exercises 1-8, find the Laplace transforms of the following functions by
showing the details of your steps. (a, b, c, and are constants) .
1. f (t) =
2
t c t b a + + 2. f (t) = sin t cos t 3. f (t) = t cos
2
4. f (t) =
t 3 cosh e
t
5. f (t) = ) t ( sin + 6.

'

1 x 0 for 1
1 x 0 for 0
) t ( f
7. 2 x 0 for 1 x 1 ) t ( f 8.

'

1 x 0 for 1
1 x 0 for t
) t ( f
In exercises 9 13, given F (s) = L (f), find f (t) by showing the details of your
steps. (, , and are constants) .
9.
4 s
4 s
2

10.
25 s
s 5
2

11.
4
s
1
12.
1
4
3
s
s 2

13.
2 s s
10 s
2


In exercises 14 17, find the Laplace transforms of the following functions by
showing the details of your steps. (a, b, c, and are constants) .
14. sinh t cos t 15. cosh t sin t 16. t 2 sinh e 5
t 2
17.
t
e t
2
) 1 ( +
In exercises 18 20, find the inverse Laplace transforms of the following by
showing the details of your steps.
18.
2
) 1 s (
1
+
19.
4
) 3 s (
12

20.
18 s 6 s
3
2
+ +

1.2 More on Transforms of Functions
1.2.1 Laplace Transforms of Derivatives
Theorem 1.3 ( Laplace Transform of the Derivative of f
(t))
Suppose that f (t) is a piecewise continuous real valued
Prepared by Tekleyohannes Negussie
92
Unit I Laplace Transform
function
for all t 0 and
) t ( f

t
e M

for some constants


and M,
and has a derivative ) t ( f ' that is piecewise continuous
on every
finite interval on [0, ). Then the Laplace transform of
) t ( f '
exists when s >

, and
) f ( L ' = sL (f) f (0), for all s >

.

Proof Applying integration by parts we get:

)] ( [ t f L
=

0
) ( dt t f e
st
=


+
1
]
1


0
0
) ( ) ( dt e t f s e t f
b
im
st
b
st


=
[ ] ]. [ ) 0 ( ) ( f L s f e b f
b
im
sb
+


Since
b
Me b f

) ( , we have
sb
e b f

) (

b s b
e Me b f


) ( =
b s
Me
) (
But,
b s
Me
b
im ) (


=
b s
e
M
b
im
) (

= 0, for
> s
.
This implies that
sb
e b f
b
im


) (

= 0, and hence
sb
e b f
b
im


) (

= 0.
Consequently,

) 0 ( )] ( [ )] ( [ f t f sL t f L

Therefore, ) f ( L ' = sL (f) f (0), for all s >

.
Theorem 1.4 (Laplace Transforms of Higher order
Derivatives)
Let ) (t f and its derivatives ) (t f
, ) (t f
,

, ) (
) 1 (
t f
n
be real
valued
continuous functions for all 0 t and satisfying ) (
) (
t f
k

t
e M


for some

and M, and k 0 and let the derivative ) (


) (
t f
n
be
Prepared by Tekleyohannes Negussie
93
Unit I Laplace Transform
piecewise
continuous on every finite interval contained in ) , 0 [ . Then the
Laplace
transform of ) (
) (
t f
n
exists when
> s
, and is given by
)) ( (
) (
t f L
n
= ) 0 ( ) 0 ( ) 0 ( )) ( (
) 1 ( 2 1

n n n n
f f s f s t f L s
Proof ( by the principle of mathematical induction on n)
For n = 1, it holds by theorem 1.4 i.e
) 0 ( )] ( [ )] ( [ f t f sL t f L
.
Assume that it holds true for n = k

] [
) (k
f L
=
) 0 ( ... ) 0 ( ' ) 0 ( ] [
) 1 ( 2 1

k k k
k
f f s f s f L s
Now we need to show that it holds true for n = k + 1

] [
) 1 ( + k
f L
=
) 0 ( ] [
) ( ) ( k k
f f L s

=
[ ] ) 0 ( ... ) 0 ( ' ) 0 ( ] [
) 1 ( 2 1

k k k
k
f f s f s f L s s
=
) 0 ( ... ) 0 ( ' ) 0 ( ] [
) ( 1 1 k k
k
k
f f s f s f L s
+
Therefore, by the principle of mathematical induction
)) ( (
) (
t f L
n
= ) 0 ( ) 0 ( ) 0 ( )) ( (
) 1 ( 2 1

n n n n
f f s f s t f L s
for any natural number n.
Example 14 Let . ) (
3
t t f Then find ]. [ f L
Solution ) (t f
=
2
3t
, 0 ) 0 ( f , ) (t f
= t 6 , 0 ) 0 ( f , 6 ) ( t f and . 6 ) 0 ( f
Thus, )] ( [ t f L
= ) 0 ( ) 0 ( ) 0 ( )] ( [
2 3
f f s f s t f L s , and also, )] ( [ t f L
=
s
L
6
) 6 ( .
Equating these values we get:
] [
3
f L s =
s
6

4
6
] [
s
f L
Therefore, ] [
3
t L =
4
6
s
.
Remark: It can be shown by induction on n that
) (
n
t L =
1
!
+ n
s
n
for
n
= 0, 1, 2,

and s > 0.
Example 15 Find ). (cos t L
Solution Let ) (t f = . cos t Then ) 0 ( f = 1, ) (t f
= t sin , ) 0 ( f
= 0,
) (t f
=
t cos
2

= ). (
2
t f
Prepared by Tekleyohannes Negussie
94
Unit I Laplace Transform
Now we have )) ( ( t f L
= ) 0 ( ) 0 ( )) ( (
2
f sf t f L s and )] ( [ t f L
= ] [
2
f L

)] ( [ t f L
=
2 2
+ s
s
.
Therefore, ) (cos t L = .
2 2
+ s
s

Example 16 Find L [
t
2
cos
].
Solution Let ) (t f =
t
2
cos
. Then f (0) = 1, ) (t f
= t t sin cos 2 , ) 0 ( f
= 0 and
) (t f
=
t t
2 2
cos 2 sin 2
= ] cos [sin 2
2 2
t t
= ] cos cos 1 [ 2
2 2
t t =
t
2
cos 4 2
= ). ( 4 2 t f
Now, since )) ( ( t f L
= ), 0 ( ) 0 ( )) ( (
2
f sf t f L s
we have
] 2 ) ( 4 [ + t f L = ) 0 ( ) 0 ( )) ( (
2
f sf t f L s and ] 2 ) ( 4 [ + t f L = ) 2 ( )] ( [ 4 L t f L +
s f L s ) (
2
=
) 2 ( ] [ 4 L f L +
L[f] =
) 4 (
2
2
2
+
+
s s
s
.
Therefore, ) (cos
2
t L =
) 4 (
2
2
2
+
+
s s
s
for s > 0.
Example 17 Show that ) (sin
2
t L =
) 4 (
2
2
+ s s
.
Solution Let f (t) = ) (sin
2
t L . Then f (0) = 0, ) (t f
= t t sin cos 2 , ) 0 ( f
= 0 and
) (t f
=
t t
2 2
sin 2 cos 2
= ] sin 2 1 [ 2
2
t = ) ( 4 2 sin 4 2
2
t f t .
Now, since )) ( ( t f L
= ), 0 ( ) 0 ( )) ( (
2
f sf t f L s
we have: ] 2 ) ( 4 [ + t f L = ) 0 ( ) 0 ( )) ( (
2
f sf t f L s and ] 2 ) ( 4 [ + t f L = ) 2 ( )] ( [ 4 L t f L +
) (
2
f L s =
) 2 ( ] [ 4 L f L +
L[f] =
) 4 (
2
2
+ s s
.
Therefore, ) (sin
2
t L =
) 4 (
2
2
+ s s
for s > 0.
Example 18 Find ) cos ( t t L .
Solution Let ) (t f =
t t cos
. Then f (0) = 0, ) (t f
= t t t sin cos , ) 0 ( f
= 1
and ) (t f
= t t t t cos sin sin
2
= ). ( sin 2
2
t f t
Thus, )] ( [ t f L
= )] ( sin 2 [
2
t f t L
) 0 ( ) 0 ( )) ( (
2
f sf t f L s = )]. ( [ ) (sin 2
2
t f L t L
1 ) (
2
f L s = ]. [ 2
2
2 2
f L
s

Prepared by Tekleyohannes Negussie


95
Unit I Laplace Transform
) ( ) (
2 2
f L s + = 1
2
2 2
2
+
+

= .
2 2
2 2

s
s

Therefore, ) cos ( t t L = )] ( [ t f L =
2 2
2 2
) (
2

s
s

Similarly, it can be shown that
) cos ( t t L =
2 2 2
2 2
) (

s
s
and ) sin ( t t L =
2 2 2
) (
2

+ s
s
.
1.2.2 Laplace Transforms of the Integral
Theorem 1.5 (Integration of f (t))
Let F (s) be the Laplace transform of the real valued
function
f (t). If f (t) is piecewise continuous and satisfies the
inequality

t
e M t f

) (
for some constants and M, then
) (
1
) (
0
s F
s
d f L
t

'


for s > 0 and s > .
Proof Suppose that f (t) is piecewise continuous and
t
e M t f

) ( for some
constants
and M . Then the integral
g (x) =

t
d f
0
) (
is continuous and for any positive number t

t
dt f t g
0
) ( ) (

t
d e M
0


= ) 1 (

e
k
M


e
k
M
for s > .
This shows that g (t) also satisfies an inequality of the form
) (t g

e
k
M
for s > .
Also, ) ( ' t g = f (t), except for points at which f (t) is discontinuous. Hence,
) ( ' t g is
piecewise continuous on each finite interval, and by theorem 1.4,
Prepared by Tekleyohannes Negussie
96
Unit I Laplace Transform
L[f (t)] = )] ( [ ' t g L = s L [g (t)] g (0) for s > .
Hence, g (0) = 0, so that L [f (t)] = s L [g (t)] for s > 0 and s > .
Therefore, ) (
1
) (
0
s F
s
d f L
t

'

for s > 0 and s > .


Example 19 Let F (s) =
) (
1
2 2
+ s s
. Find f (t).
Solution From the result of example 7 we get

1
]
1

) (
1
2 2
1
s s
L = t

sin
1
Again, using theorem 1.7 we have

1
]
1

) (
1
2 2
1
s s
L =

d
t

0
sin
1
= ) cos 1 (
1
2
t


Therefore, f (t) = ) cos 1 (
1
2
t

.
Example 20 Let F (s) =
) (
1
2 2 2
+ s s
. Find f (t).
Solution From the result of example 19 we get

1
]
1

) (
1
2 2 2
1
s s
L =

d
t


0
2
) cos 1 (
1
= )
sin
(
1
2

t
t
Therefore, f (t) = )
sin
(
1
2

t
t .
1.2.3 S-shifting, Unit step functions and t-shifting
The first shifting theorem
If we replace s by s a, in the definition of the Laplace transform we get the
following important result.
Theorem 1.6 (First shifting theorem)
If f (t) has the transform F (s) where s > k, then ) t ( f e
at
Prepared by Tekleyohannes Negussie
1 1
97
Unit I Laplace Transform
has the
transform F (s a) where s a > k. Symbolically,
L { ) t ( f e
at
} = F (s a) or ) t ( f e
at
= )} a s ( F { L
1


Proof F (s a) can be obtained by replacing s by s a in definition 1.1 as
follows:
F (s a) =



b
0
) t ( f
t ) a s (
dt e
b
im
=
[ ]



b
0
) t ( f
t a
e
t s
dt e
b
im
= L { ) t ( f e
at
}.
Now if F (s) exists for s greater than some k, then our integral exists for
s a > k.
Example 21 Find the Laplace transform of f (t) =
at
e cos t and g (t) =
at
e sin t
for t 0.
Solution Applying theorem 1.6 on the result of example 5 we get:
L (f ) = L (
at
e cos t) =
2 2
) a s (
a s
+

and L (g ) = L (
at
e sin t) =
2 2
) a s ( +


Unit step functions and t-shifting Theorem
The unit step function defined below is a typical engineering function made to
measure for engineering applications, which often involve function that are off
and on.

Definition 1.3 The function u
defined by

'

>
<

a t for
a t for
a t u
1
0
) (
is called the unit step
function.

The unit step function is also called the Heaviside function.
Example 22 Let f (t) = 5 sin t. Then f (t) u(t 2) represents a switch off for 0 < t
<2 and switch
on for t > 2 while f (t 2) u(t 2) represents a shift to the right by 2
units.
Prepared by Tekleyohannes Negussie
1 1
98
Unit I Laplace Transform
Example 23 Show that L [u (t a)] =
s
e
s a
for s > 0.
Solution Applying definition 1.1 we get:
L [u (t a)] =

b
t s
dt a t u e
b
im
0
) (



b
a
t s
dt e
b
im
=
b
im

a
b
s
e
s a

'

=
s
e
s a
.
Therefore, L [u (t a)] =
s
e
s a
for s > 0.

Theorem 1.7 (The second shifting theorem, t-shifting
theorem)
If f (t) has the Laplace transform F (s), then the shifted
function
) (
~
t f = f (t a) u (t a) =

'

a t for
a t for

1
0
has the Laplace transform
) (s F e
as
. That is
L[f (t a) u (t a)] =
) (s F e
as
.
Proof From the definition of the Laplace transform we have

) (s F e
as
=



b
d f e
b
im
e
s as
0
) (

=

+

b
d f e
b
im a s
0
) (
) (

Substituting + a = t in the integral we get:

) (s F e
as
=

b a
dt a t f e
b
im
a
t s
) (

b
dt a t u a t f e
b
im t s
0
) ( ) (

= L[f (t a) u (t a)].
Prepared by Tekleyohannes Negussie
t
f(t)
t
f (t)
1 1
1
f (t) = u (t 1)
1
f (t) = t [u (t) u (t 1)] + u (t 1)
99
Unit I Laplace Transform
Therefore,
) (s F e
as
= L[f (t a) u (t a)].
Example 24 Find L [f], where

'

2 sin
2 0
0 2
) (



t if t
t if
t if
t f
Solution We write f (t) in terms of unit step functions. For 0 < t < , we take 2 u
(t).
For < t < 2 we want 0, so we must subtract the step function 2 u (t
) and for
t > 2 we need to add 2 u (t ) sin t.
Hence, f (t) = 2 u (t) 2 u (t ) 2 u (t 2) sin t. f (t) = 2 u (t) 2 u (t ) 2 u
(t 2) sin (t 2). (since sine is a periodic function with period 2)
Therefore, L [f] =
s
e
s
s

2 2
+
1
2
2
+

s
e
s
.
Example 25 Find the inverse Laplace transform of
F (s) =
2
2
2
2 2
s
e
s
s

s
e
s 2
4

+
1
2
+

s
e s
s
.
Solution Without the exponential functions the four terms of F (s) would have
the inverses
2t, 2t, 4 and cos t. Hence, by theorem 1.7
f (t) = 2t 2(t 2) u(t 2) 4 u(t 2) + u (t ) cos (t )= 2t 2t u (t 2) u (t
)cos t.
Therefore,

'



t if t
t if
t if t
t f
cos
2 0
2 0 2
) ( .
1.2.3 Laplace
Transforms of
Periodic Functions

Proof By definition 1.1
Prepared by Tekleyohannes Negussie

Theorem 1.8 (Laplace Transforms of Periodic
Functions)
The Laplace transform of a piecewise
continuous
function f (t) with period p is
L [f] =

,
_

p
t s
p s
dt t f e
e
0
) (
1
1

100
Unit I Laplace Transform
L [f] =


m
n
p n
np
t s
dt t f e
m
im
0
) 1 (
) (

= ( )



0
0
) (
n
p
t s n p s
dt t f e e
m
im
(put t = t + np since f (t) is periodic with
period p)
=

,
_

p
t s
t s
dt t f e
e
0
) (
1
1
, because it is a geometric series.
Therefore, L [f] =

,
_

p
t s
t s
dt t f e
e
0
) (
1
1
.
Example 26 Find the Laplace transform of the saw-tooth wave given by

p
k
t f ) (
where 0 t p and f (t + p) = f (t) for t 0.
Solution From theorem 1.8 we have
L [f] =

,
_

,
_

p
t s
t s
dt e
e
p
k
0
1
1
=

'

,
_

,
_

0
1
1
p
s
e
e p
k
t s
t s
=
ps
k
Therefore,
1
]
1

t
p
k
L =
ps
k
.
1.3 Differential Equations
1.3.1 Ordinary Linear Differential Equations
We shall now discuss how the Laplace transform method solves differential
equations. We began with an initial value problem

) (t r by y a y + +
, y (0) =
0
k and
1
) 0 ( k y

(1)
with constants a and b. Here r (t) is the input (driving force) applied to the
mechanical system and y (t) is the output (response of the system). In Laplace
method we do three steps:
Step 1 We transfer (1) by means of theorem 1.3 and 1.4, writing Y = L (y) and R
= L (r). This gives

[ ] [ ] ) ( ) 0 ( ) 0 ( ) 0 (
2
s R bY y sY a y y s Y s + +

(2)
This is called the subsidiary equation.
Collecting Y terms we have
Prepared by Tekleyohannes Negussie
101
Unit I Laplace Transform
( ) ( ) ) ( ) 0 ( ) 0 (
2
s R y y a s Y b as s + + + + +
(3)
Step II We solve the subsidiary equation algebraically for Y. Division by
b as s + +
2
and use
of the so- called the transfer function

b as s
s Q
+ +

2
1
) (

(4)
gives the solution
( ) [ ] ) ( ) ( ) 0 ( ) 0 ( ) ( s Q s Q y y a s Y s R + + +
(5)
If y (0) = ) 0 ( y
= 0, then this implies Y = QR; thus Q is the quotient

R
Y
s Q ) ( =
) (
(
input L
output L

(6)
and this explains the name of Q.
Note that: Q depends only on a and b, but neither on r (t) nor on the
initial conditions.
Step III We reduce (5) to a sum of terms whose inverse can be found from the
table, so that
the solution
y (t) =
] [
1
Y L

.
Example 27 (initial value problem)
Solve
t y y
, y (0) = 1 and ) 0 ( y
= 1
Solution The subsidiary equation becomes

2
1
) 0 ( ) 0 (
2
s
Y y y s Y s
Thus,
2
1
1 ) 1 (
2
s
s Y s +
( ) 1
1
1
1
2 2 2

+

s s s
s
Y

Prepared by Tekleyohannes Negussie
102
Unit I Laplace Transform

( ) 1
1
1
1
2 2

+
+

s s s
Y
=
1
]
1

+
+

2
1
1
1
1
1
2
s s s

Now
t
e
s
L

1
]
1

+1
1 1
, t
s
L sinh
1
1
2
1

1
]
1

and t
s
L
1
]
1


2
1 1
.
Therefore, y (t) = t
t
e

+ sinh t.

Example 28 (initial value problem)
Solve
t
e y y y

+ + 2
, y (0) = 1 and ) 0 ( y
= 1
Solution The subsidiary equation becomes

( ) ( )
1
1
1 2 1
2
+
+ + + +
s
Y sY s Y s

Thus,
1
1
1 ) 1 2 (
2
+
+ + +
s
s Y s s

3
) 1 (
1
) 1 (
1
2
+
+
+
+

s s
s
Y


3
) 1 (
1
1
1
+
+
+

s s
Y

Now
t
e
s
L

1
]
1

+1
1 1
,
2
3
1
2
1
) 1 (
1
t
s
L
1
]
1

t
e

.
Therefore, y (t) =
t
e t

,
_

1
2
1
2
.
Example 29 (initial value problem)
Solve
t
e t y y y
3
4 2 3 + +
, y (0) = 1 and ) 0 ( y
= 1
Solution The subsidiary equation becomes

( ) ( )
3
1 4
2 1 3 1
2
2

+ + +
s s
Y sY s Y s

Thus,
3
1 4
4 ) 2 3 (
2
2

+ + +
s s
s Y s s


( ) ( ) 2 3 ) 3 (
1
2 3
4
2 3
4
2 2 2 2
+
+
+
+
+

s s s s s s s s
s
Y

Now
2 1 2 3
4
2

+

s
B
s
A
s s
s
Y
A + B = 1 and 2A + B = 4 A = 3 and
B = 2

( ) 2 3
4
2 2
+ s s s
=
s
A
+
2
s
B
+
1 s
C
+
2 s
D
A = 3, B = 2, C = 4 and D =
1
Prepared by Tekleyohannes Negussie
103
Unit I Laplace Transform
and
( ) 2 3 ) 3 (
1
2
+ s s s
=
3 s
A
+
1 s
B
+
2 s
C
A = 0.5, B = 0.5 and C = 1.
Hence,
( ) ( ) 2 3 ) 3 (
1
2 3
4
2 3
4
2 2 2 2
+
+
+
+
+

s s s s s s s s
s
=
s
3
+
2
2
s

,
_

,
_

1
1
2
1
s

2
2
s
+

,
_

,
_

3
1
2
1
s

Now 3
3 1

1
]
1


s
L , t
s
L 2
2
2
1

1
]
1


,
1
]
1

,
_

,
_


1
1
2
1 1
s
L =
t
e
2
1
and
1
]
1

,
_

,
_


3
1
2
1 1
s
L =
t
e
3
2
1
.
Therefore, y (t) = 3 + 2t
t
e
2
1

t
e
2
2 +
t
e
3
2
1
.
Example 30 (initial value problem)
Solve
t
t
e y y y sin 5 2

+ +
, y (0) = 0 and ) 0 ( y
= 1.
Solution The subsidiary equation becomes

( ) ( )
1 ) 1 (
1
5 2 1
2
2
+ +
+ +
s
Y sY Y s

Thus,
( )
1 ) 1 (
1
1 4 ) 1 (
2
2
+ +
+ + +
s
Y s

( )( ) 4 ) 1 ( 1 ) 1 (
1
4 ) 1 (
1
2 2 2
+ + + +
+
+ +

s s s
Y
Now
( )( ) 4 ) 1 ( 1 ) 1 (
1
2 2
+ + + + s s
=
4 ) 1 (
2
+ +
+
s
B As
+
1 ) 1 (
2
+ +
+
s
D Cs
1 ) 5 2 ( ) 2 5 2 2 ( ) 2 2 ( ) (
2 3
+ + + + + + + + + + + D B s D C B A s D C B A s C A
A = C = 0, B =
3
1
and D =
3
1
.
Hence, Y =
1
]
1

+ +

+ +

,
_

+
+ + 4 ) 1 (
1
1 ) 1 (
1
3
1
4 ) 1 (
1
2 2 2
s s s
=
1
]
1

+ +
+
+ +

,
_

4 ) 1 (
2
1 ) 1 (
1
3
1
2 2
s s
Now
1
1
]
1

+ +

4
2
) 1 (
2 1
s
L = t
t
e 2 sin

and
1
1
]
1

+ +

1
2
) 1 (
1 1
s
L = t
t
e sin

Therefore, y (t) =
t
e t t

+ ) 2 sin sin (
3
1
.
Example 31 (initial value problem)
Solve
t y y y sin 10 2 2 + +
, y (0) = 0 and ) 0 ( y
= 1.
Solution The subsidiary equation becomes
Prepared by Tekleyohannes Negussie
104
Unit I Laplace Transform

( )
1
10
2 2 1
2
2
+
+ +
s
Y sY Y s

Thus,
( )
1
10
1 1 ) 1 (
2
2
+
+ + +
s
Y s

( )( ) 1 1 ) 1 (
10
1 ) 1 (
1
2 2 2
+ + +
+
+ +

s s s
Y
Now
( )( ) 1 1 ) 1 (
10
2 2
+ + + s s
=
1 ) 1 (
2
+ +
+
s
B As
+
1
2
+
+
s
D Cs
10 ) 2 ( ) 2 2 ( ) 2 ( ) (
2 3
+ + + + + + + + + D B s D C A s D C B s C A
A = 4, B = 6, C = 4 and D = 2.
Hence, Y =
1
]
1

+
+
1
]
1

+ +
+
+
+ + 1
1
2
1 1 ) 1 (
1
4
1 ) 1 (
3
2 2 2 2
s s
s
s
s
s
Now
1
1
]
1

1
2
1 1
s
L =
t sin
and
1
1
]
1

+ +
+
1
2
) 1 (
1 1
s
s
L = t
t
e sin

Therefore, y (t) =
t
e t t t t

+ + ) cos 4 sin 3 ( cos 4 sin 2 .
Example 32 (initial value problem)
Solve
t y y cosh 18 9 +
, y (0) = 2 and ) 0 ( y
= 0.
Solution The subsidiary equation becomes

( )
1
18
9 2
2
2

+
s
s
Y s Y s

Thus,
( )
1
18
2 9
2
2

+ +
s
s
s Y s

( )( ) 1 9
18
9
2
2 2 2
+
+
+

s s
s
s
s
Y
Now
( )( ) 1 9
18
2 2
+ s s
s
=
9
2
+
+
s
B As
+
1
2

+
s
D Cs
s D B s C A s D B s C A 18 ) ( ) 9 ( ) ( ) (
2 3
+ + + + + + +
B = D = 0, A =
5
9
and B =
5
9
.
Hence, Y =
1
]
1

,
_

+
+ 9 1 5
9
9
2
2 2 2
s
s
s
s
s
s
=
1
]
1

+
+

,
_

9 1
9
5
1
2 2
s
s
s
s
Now
1
1
]
1

1
2
1
s
s
L =
t sh co
and
1
1
]
1

9
2
1
s
s
L =
t 3 cos
Therefore, y (t) = t t 3 cos
5
1
cosh
5
9
+ .
Example 33 (shifted data problem)
Solve
t y y 2 ' '
1
+
,
2 4

,
_

y and 2 2
4
'

,
_


y .
Prepared by Tekleyohannes Negussie
105
Unit I Laplace Transform
Solution set t =
0
~
t t + , so that
4
0

t .
Now the subsidiary equation in terms of the new variable t
~
becomes

s s
Y y y s Y s
2
2 ~
) 0 ( '
~
) 0 (
~
~
2
2

+ +

s s
Y s Y s
2
2 ~
2 2
2
~
2
2

+ + +

s s
s Y s
2
2
2 2
2
~
) 1 (
2
2

+ + + +

) 1 ( 2 ) 1 (
2
1
2
1
2
) 1 ( 2
~
2 2 2 2 2 2
+
+
+
+
+

+
+
+

s s s s s s s
s
Y

Now
) 1 (
2
2 2
+ s s
=
1
]
1

1
1 1
2
2 2
s s
and
) 1 ( 2
2
+ s s

=
1
]
1

+

,
_

1
1
2
2
s
s
s

Hence,
1
2
1
2
) 1 ( 2
~
2 2 2
+

+
+
+

s s s
s
Y

+
1
]
1

1
1 1
2
2 2
s s
+
1
]
1

+

,
_

1
1
2
2
s
s
s

=
1
2
2
+

s
+
2
2
s
+
,
_

s 2

Therefore, y
~
( t
~
) = t
~
sin 2 + 2 t
~
+
2

.
Now in terms of the original variable t we get:
y(t) = 2t + cos t sin t.
Therefore, y(t) = 2t + cos t sin t.
1.3.2 Systems of Linear Differential Equations
Example 34 Solve
2 1 1
' y y y +
,
2 1 2
' y y y
,
1 ) 0 (
1
y
and
0 ) 0 (
2
y
.
Solution The subsidiary equations become

2 1 1 1
) 0 ( Y Y y sY + and
2 1 2 2
) 0 ( Y Y y sY 1 ) 1 (
2 1
+ Y Y s and
0 ) 1 (
1 2
+ + Y Y s
Solving for
1
Y and
2
Y algebraically we get
) 1 ( ) 1 ( ) 1 (
2 1
2
+ + + s Y s Y s and 0 ) 1 (
1 2
+ + Y Y s
[ ] ) 1 ( 1 ) 1 (
1
2
+ + + s Y s and
1 2
1
1
Y
s
Y

,
_

+


1 ) 1 (
1
2 1
+ +
+

s
s
Y
and
1 ) 1 (
1
2 2
+ +

s
Y
Prepared by Tekleyohannes Negussie
106
Unit I Laplace Transform
Now,
1
]
1

+ +
+
1 ) 1 (
1
2
1
s
s
L =
t
e

cos t and
1
]
1

+ +

1 ) 1 (
1
2
1
s
L =
t
e

sin t
Therefore, t
t
e t y cos
1
) (

and t
t
e t y sin
2
) (

.
Example 35 Solve
2 1 1
5 ' y y y +
,
2 1 2
5 ' y y y +
,
3 ) 0 (
1
y
and
7 ) 0 (
2
y
.
Solution The subsidiary equations become

2 1 1 1
5 ) 0 ( Y Y y sY + and
2 1 2 2
5 ) 0 ( Y Y y sY +

3 ) 5 (
2 1
Y Y s
and
7 ) 5 (
1 2
Y Y s
Solving for
1
Y and
2
Y algebraically we get
) 5 ( 3 ) 5 ( ) 5 (
2 1
2
s Y s Y s and 7 ) 5 (
1 2
Y Y s
[ ] ) 5 ( 3 7 1 ) 5 (
1
2
s Y s and
1 2
5
1
5
7
Y
s s
Y

,
_


1
]
1

1 ) 5 (
5
3
1 ) 5 (
7
2 2 1
s
s
s
Y
and
( )
1
]
1

1 ) 5 (
3
5
7
1 ) 5 ( ) 5 (
7
2 2 2
s s s s
Y
Now
( ) 5
7
1 ) 5 (
5
7
1 ) 5 ( ) 5 (
7
2 2

1
]
1

s s
s
s s
.
Hence,
1
]
1

1
]
1

1 ) 5 (
3
1 ) 5 (
5
7
2 2 2
s s
s
Y .
Now,
1
]
1



1 ) 5 (
5
2
1
s
s
L =
t
e
5
cosh t and
1
]
1

1 ) 5 (
1
2
1
s
L =
t
e
5
sinh t
Therefore, ) cosh 3 sinh 7 (
5
1
) ( t t
t
e t y and ) sinh 3 cosh 7 (
5
2
) ( t t
t
e t y .
Example 36 Solve
t y y 2 cos 5 ' '
2 1
+
,
t y y 2 cos 5 ' '
1 2
+
,
1 ) 0 (
1
y
,
1 ) 0 ( '
1
y
,
1 ) 0 (
2
y
and
1 ) 0 ( '
2
y
.
Solution The subsidiary equations become

4
5
) 0 ( ' ) 0 (
2 2 1 1 1
2
+
+
s
s
Y y y s Y s
and
4
5
) 0 ( ' ) 0 (
2 1 2 2 2
2
+
+
s
s
Y y y s Y s

( ) 4
5
1
2 2 1
2
+
+ +
s
s
s Y Y s
and
4
5
1
2 1 2
2
+
+ +
s
s
s Y Y s
Solving for
1
Y and
2
Y algebraically we get

( ) 4
5
2
3
3 2
2
2
1
4
+
+ +
s
s
s s Y s Y s and
4
5
1
2 1 2
2
+
+
s
s
s Y Y s
Prepared by Tekleyohannes Negussie
107
Unit I Laplace Transform

4
5
4
5
1 ) 1 (
2 2
3
3 2
1
4
+

+
+ + +
s
s
s
s
s s s Y s and
( )
1
2
2 2
4
5
1 Y s
s
s
s Y
+
+

1
]
1

,
_

+
+

4
5
) 1 (
1
1 1
1
2 2 2 2 1
s
s
s s
s
s
Y

Now
4 1 ) 4 )( 1 (
5
2 2 2 2
+
+
+

+ s
D Cs
s
B As
s s
s

4 1 ) 4 )( 1 (
5
2 2 2 2
+

+ s
s
s
s
s s
s

Hence,
4 1
1
2 2 1
+
+
+

s
s
s
Y
and
4 1
1
2 2 2
+

s
s
s
Y
Therefore, t t t y 2 cos sin ) (
1
+ and t t t y 2 cos sin ) (
2
.
Example 37 Solve
2 1 1
3 ' ' y y y +
,
t
e y y 4 4 ' '
1 2
,
2 ) 0 (
1
y
,
3 ) 0 ( '
1
y
,
1 ) 0 (
2
y
and
2 ) 0 ( '
2
y
.
Solution The subsidiary equations become

2 1 1 1 1
2
3 ) 0 ( ' ) 0 ( Y Y y y s Y s + and
1
4
4 ) 0 ( ' ) 0 (
1 2 2 2
2


s
Y y y s Y s
s Y Y s 2 3 3 ) 1 (
2 1
2
+ and
1
4
2 4
1 2
2

+
s
s Y Y s
Solving for
1
Y and
2
Y algebraically we get
s Y Y s 8 12 12 ) 1 ( 4
2 1
2
+ and
) 1 ( 4 ) 1 ( ) 2 ( ) 1 ( 4 ) 1 (
2
1
2
2
2 2
+ + s s s Y s Y s s

2
1
2

s
Y
and
2
1
1
1
1

s s
Y
Therefore,
t t
e e t y
2
1
) ( + and
t
e t y
2
2
) ( .
Prepared by Tekleyohannes Negussie
108

Vous aimerez peut-être aussi